LSAT and Law School Admissions Forum

Get expert LSAT preparation and law school admissions advice from PowerScore Test Preparation.

 Administrator
PowerScore Staff
  • PowerScore Staff
  • Posts: 8950
  • Joined: Feb 02, 2011
|
#23878
Complete Question Explanation

Cannot Be True—CE. The correct answer choice is (D)

This stimulus deals with the effects of schizophrenia on the brain. When identical twins, alike in most other ways, include one schizophrenic, the effected twin has areas of the brain that are smaller. The author’s (questionable) conclusion: schizophrenia is caused by damage to the brain.

The question stem asks this: if the stimulus’ premises are all true, which answer choice cannot be true. Among the five answer choices, four could be true based on the information in the stimulus, and one cannot be true.

Answer choice (A): This is possible; even thought the author concludes that the disease is caused by damage, one might require a susceptibility to be a candidate.

Answer choice (B): Since the stimulus provides no information regarding prospective cures, this could be true, so this answer choice cannot be correct.

Answer choice (C): This is possible based on the information in the stimulus, so this answer choice is incorrect.

Answer choice (D): This is the correct answer choice. If, as the stimulus provides, the brains of identical twins are identical, then genetic information alone would not allow one to distinguish two identical twins (or to tell which one will develop schizophrenia).

Answer choice (E): This is possible (and consistent with the author’s conclusion), so this cannot be the right answer to this cannot be true question.
 kim4956
  • Posts: 18
  • Joined: Nov 25, 2015
|
#20768
The brains of identical twins are genetically identical. When only one of a pair of identical twins is a schizophrenic...

I don't understand why (B) is the correct answer. I can see why some of the answer choices (C, E, and maybe even A) are incorrect, but have a hard time seeing how (B) is the best answer choice... Could you explain the reasoning?

Thank you!
 Steve Stein
PowerScore Staff
  • PowerScore Staff
  • Posts: 1153
  • Joined: Apr 11, 2011
|
#20769
Hi Kim,

The correct answer to that one is actually answer choice (D) :-D

Please let me know whether that makes more sense—thanks!

~Steve
 kim4956
  • Posts: 18
  • Joined: Nov 25, 2015
|
#20771
Sorry! I meant (D)... Huge typo there.

Care to explain why that's the correct answer? :)
 Steve Stein
PowerScore Staff
  • PowerScore Staff
  • Posts: 1153
  • Joined: Apr 11, 2011
|
#20778
Hey Kim,

Thanks for your response. The question in that one specifies that all of the answer choices could be true except for one—as you know, this tells us that the correct answer choice will be the one that cannot be true.

Answer choice (D) presents the claim that it will eventually be possible to predict schizophrenia based on genetic information alone. The stimulus provides that the brains of identical twins are genetically identical, and yet in some cases only one twin develops the disease, and the other doesn't; this tells us that genetic information alone cannot be sufficient to make such a determination.

I hope that's helpful! Please let me know whether this is clear—thanks!

~Steve
 kim4956
  • Posts: 18
  • Joined: Nov 25, 2015
|
#20831
Thanks, Steve!
 pepitohonguito
  • Posts: 1
  • Joined: Feb 02, 2018
|
#45257
What about if in the future they can determine by a blood work using your genetic information that you got some brain damage and that's the root of your schizophrenia?

Answer A mentions genetic susceptibility not physical susceptibility so I can't understand why it can be true.
 Alex Bodaken
PowerScore Staff
  • PowerScore Staff
  • Posts: 135
  • Joined: Feb 21, 2018
|
#45259
Pepitohonguito,

Let me see if I can help a bit. I want to start by saying this is a pretty tricky question (as might be expected, it's #24 in the section after all) as most "could be true EXCEPT" questions are. This means, as you know, four answers could be true while one (credited) answer cannot be true.

I want to address your comment in two ways. First, I want to explain why (D) is the credited answer, in other words, why it cannot be true. Then, I'll explain why (A) is not credited, i.e. why it could be true.

The question reads: "If the statements on which the conclusion above is based are all true, each of the following could be true EXCEPT..." It's clear: we must accept each premise in the stimulus as true (we normally do this anyhow, but the question stem is a nice reminder). So what are these premises? Taken directly from the stimulus, they are:

1.) The brains of identical twins are genetically identical.
2.) When only one of a pair of identical twins is a schizophrenic, certain areas of the affected twin's brain are smaller than corresponding areas in the brain of the unaffected twin.
3.) No such differences arc found when neither twin is schizophrenic (i.e., if neither twin is schizophrenic, certain areas of the affected twin's brain are NOT smaller than corresponding areas in the brain of the unaffected twin).

So we are looking for an answer that CANNOT be true - i.e., that, one that is made incompatible with the above statements. Well, look at (D): "It will eventually be possible to determine whether or not someone will develop schizophrenia on the basis of genetic information alone." This is directly at odds with statement (1) above. If the brains of identical twins are genetically identical, scientists will never be able to determine whether someone will develop schizophrenia on genetics alone, no matter what developments are made. The only way they could do that in the future is if they discovered somehow that twins weren't genetically identical - but we have to assume the truth of the statements, which rules that out.

As for answer choice (A), it reads "People who lack a genetic susceptibility for the disease will not develop schizophrenia." This could be true. If it were true, it would imply that among the sets who lack genetic susceptibility, no twins will develop schizophrenia, and among sets who do not lack genetic susceptibility (i.e. have a genetic susceptibility), either twin or both could get schizophrenia, depending upon the size of the certain areas of the brain mentioned in the question. None of this conflicts with the statements above.

I hope this was helpful, and feel free to follow up with us if you have more questions!

Alex
 Jude.m.stone@gmail.com
  • Posts: 18
  • Joined: Mar 12, 2023
|
#102159
Hi Alex, thanks for explaining why D is correct and A isn't. The part that's still tripping me up about eliminating A is that based on what we've learned about causal reasoning, when an LSAT author states a cause and effect, there is only one cause and if the cause happens, the effect MUST happen too (do I have that right?).

So my thought process is that if A says people who don't have a genetic predisposition for schizophrenia won't get it, the contrapositive is that if you do get schizophrenia then you must be genetically susceptible. To me, that's at odds with the stimulus, which states that the cause -- the ONLY cause, because it's in the LSAT world -- is physical brain structure damage. By having a genetic predisposition be a requirement, doesn't that make it another contributing factor or cause? Because my logic was that if A is suggesting that genetics play a role and not solely physical damage (i.e. what if someone's brain is damaged but they're not genetically predisposed? Wouldn't A suggest they wouldn't be schizophrenic?), then it violates the causal relationship in the stimulus. Please help me see my error so I can avoid this mistake in the future!
User avatar
 Jeff Wren
PowerScore Staff
  • PowerScore Staff
  • Posts: 705
  • Joined: Oct 19, 2022
|
#102173
Hi Jude,

The easiest way to rule out Answer A is by carefully reading/understanding the question stem.

The question stem asks, "If the statements on which the conclusion above is based are all true, each of the following could be true EXCEPT."

The wording here is easy to misread or overlook, but the conclusion is not included in the statements that we are to accept as true, only the premises (i.e. the statements on which the conclusion is based).

The statement about schizophrenia being caused by damage to the physical structure of brain only appears in the conclusion and is therefore excluded for the purpose of this question.

Get the most out of your LSAT Prep Plus subscription.

Analyze and track your performance with our Testing and Analytics Package.